A 1-form on a smooth manifold is exact if and only if it integrates to zero on every closed curve











up vote
5
down vote

favorite
2












I am stuck on the following problem, which comes from a old qualifying exam.



Prove that a 1-form $phi$ on $M$ is exact if and only if for every closed curve
c, $int_{c} phi =0$.



One way is an application of Stokes' theorem,
if $phi = df$ then $int_{c}phi = int_{partial C}df = 0$ since $partial B=emptyset$.



I don't know how to do the other direction. I made an attempt as follows:



Choose any $x_0 in M$ define a function, $f(x)=int_{x_0}^{x}phi$. This makes sense since the integral is path independent. Now I want to prove that $f$ is smooth and $df=phi$. I can't do either.



Thanks










share|cite|improve this question
























  • You may not be able to prove $f$ is smooth -- in general, it's not -- but it's once-differentiable. The proof looks a lot like the proof of the fundamental theorem of calculus (of which this is just a generalization). Same goes for $df = phi$.
    – John Hughes
    Jul 15 '14 at 21:18










  • Is the theorem true? Do I need extra requirements?
    – Bates
    Jul 15 '14 at 21:28










  • I think I can do it, if I assume that $H^1 =0$.
    – Bates
    Jul 15 '14 at 21:29






  • 7




    You don't need to assume $H^1=0$. But to prove $f$ is smooth, you have to assume that $phi$ is smooth. To prove that $df=phi$ at a point $p$, you can work in coordinates centered at $p$, in which case what you need to show is that $partial f/partial x^i(p) = phi_i(p)$ for each $i$, where the coordinate representation of $phi$ is $sum_i phi_i dx^i$. You can do this by carefully choosing a path whose last part travels along the $x^i$-axis in these coordinates. The proof is carried out in detail in my Introduction to Smooth Manifolds (2nd ed.), Theorem 11.42.
    – Jack Lee
    Jul 15 '14 at 22:19

















up vote
5
down vote

favorite
2












I am stuck on the following problem, which comes from a old qualifying exam.



Prove that a 1-form $phi$ on $M$ is exact if and only if for every closed curve
c, $int_{c} phi =0$.



One way is an application of Stokes' theorem,
if $phi = df$ then $int_{c}phi = int_{partial C}df = 0$ since $partial B=emptyset$.



I don't know how to do the other direction. I made an attempt as follows:



Choose any $x_0 in M$ define a function, $f(x)=int_{x_0}^{x}phi$. This makes sense since the integral is path independent. Now I want to prove that $f$ is smooth and $df=phi$. I can't do either.



Thanks










share|cite|improve this question
























  • You may not be able to prove $f$ is smooth -- in general, it's not -- but it's once-differentiable. The proof looks a lot like the proof of the fundamental theorem of calculus (of which this is just a generalization). Same goes for $df = phi$.
    – John Hughes
    Jul 15 '14 at 21:18










  • Is the theorem true? Do I need extra requirements?
    – Bates
    Jul 15 '14 at 21:28










  • I think I can do it, if I assume that $H^1 =0$.
    – Bates
    Jul 15 '14 at 21:29






  • 7




    You don't need to assume $H^1=0$. But to prove $f$ is smooth, you have to assume that $phi$ is smooth. To prove that $df=phi$ at a point $p$, you can work in coordinates centered at $p$, in which case what you need to show is that $partial f/partial x^i(p) = phi_i(p)$ for each $i$, where the coordinate representation of $phi$ is $sum_i phi_i dx^i$. You can do this by carefully choosing a path whose last part travels along the $x^i$-axis in these coordinates. The proof is carried out in detail in my Introduction to Smooth Manifolds (2nd ed.), Theorem 11.42.
    – Jack Lee
    Jul 15 '14 at 22:19















up vote
5
down vote

favorite
2









up vote
5
down vote

favorite
2






2





I am stuck on the following problem, which comes from a old qualifying exam.



Prove that a 1-form $phi$ on $M$ is exact if and only if for every closed curve
c, $int_{c} phi =0$.



One way is an application of Stokes' theorem,
if $phi = df$ then $int_{c}phi = int_{partial C}df = 0$ since $partial B=emptyset$.



I don't know how to do the other direction. I made an attempt as follows:



Choose any $x_0 in M$ define a function, $f(x)=int_{x_0}^{x}phi$. This makes sense since the integral is path independent. Now I want to prove that $f$ is smooth and $df=phi$. I can't do either.



Thanks










share|cite|improve this question















I am stuck on the following problem, which comes from a old qualifying exam.



Prove that a 1-form $phi$ on $M$ is exact if and only if for every closed curve
c, $int_{c} phi =0$.



One way is an application of Stokes' theorem,
if $phi = df$ then $int_{c}phi = int_{partial C}df = 0$ since $partial B=emptyset$.



I don't know how to do the other direction. I made an attempt as follows:



Choose any $x_0 in M$ define a function, $f(x)=int_{x_0}^{x}phi$. This makes sense since the integral is path independent. Now I want to prove that $f$ is smooth and $df=phi$. I can't do either.



Thanks







integration manifolds differential-forms






share|cite|improve this question















share|cite|improve this question













share|cite|improve this question




share|cite|improve this question








edited Jul 16 '14 at 2:22







user147263

















asked Jul 15 '14 at 20:57









Bates

493313




493313












  • You may not be able to prove $f$ is smooth -- in general, it's not -- but it's once-differentiable. The proof looks a lot like the proof of the fundamental theorem of calculus (of which this is just a generalization). Same goes for $df = phi$.
    – John Hughes
    Jul 15 '14 at 21:18










  • Is the theorem true? Do I need extra requirements?
    – Bates
    Jul 15 '14 at 21:28










  • I think I can do it, if I assume that $H^1 =0$.
    – Bates
    Jul 15 '14 at 21:29






  • 7




    You don't need to assume $H^1=0$. But to prove $f$ is smooth, you have to assume that $phi$ is smooth. To prove that $df=phi$ at a point $p$, you can work in coordinates centered at $p$, in which case what you need to show is that $partial f/partial x^i(p) = phi_i(p)$ for each $i$, where the coordinate representation of $phi$ is $sum_i phi_i dx^i$. You can do this by carefully choosing a path whose last part travels along the $x^i$-axis in these coordinates. The proof is carried out in detail in my Introduction to Smooth Manifolds (2nd ed.), Theorem 11.42.
    – Jack Lee
    Jul 15 '14 at 22:19




















  • You may not be able to prove $f$ is smooth -- in general, it's not -- but it's once-differentiable. The proof looks a lot like the proof of the fundamental theorem of calculus (of which this is just a generalization). Same goes for $df = phi$.
    – John Hughes
    Jul 15 '14 at 21:18










  • Is the theorem true? Do I need extra requirements?
    – Bates
    Jul 15 '14 at 21:28










  • I think I can do it, if I assume that $H^1 =0$.
    – Bates
    Jul 15 '14 at 21:29






  • 7




    You don't need to assume $H^1=0$. But to prove $f$ is smooth, you have to assume that $phi$ is smooth. To prove that $df=phi$ at a point $p$, you can work in coordinates centered at $p$, in which case what you need to show is that $partial f/partial x^i(p) = phi_i(p)$ for each $i$, where the coordinate representation of $phi$ is $sum_i phi_i dx^i$. You can do this by carefully choosing a path whose last part travels along the $x^i$-axis in these coordinates. The proof is carried out in detail in my Introduction to Smooth Manifolds (2nd ed.), Theorem 11.42.
    – Jack Lee
    Jul 15 '14 at 22:19


















You may not be able to prove $f$ is smooth -- in general, it's not -- but it's once-differentiable. The proof looks a lot like the proof of the fundamental theorem of calculus (of which this is just a generalization). Same goes for $df = phi$.
– John Hughes
Jul 15 '14 at 21:18




You may not be able to prove $f$ is smooth -- in general, it's not -- but it's once-differentiable. The proof looks a lot like the proof of the fundamental theorem of calculus (of which this is just a generalization). Same goes for $df = phi$.
– John Hughes
Jul 15 '14 at 21:18












Is the theorem true? Do I need extra requirements?
– Bates
Jul 15 '14 at 21:28




Is the theorem true? Do I need extra requirements?
– Bates
Jul 15 '14 at 21:28












I think I can do it, if I assume that $H^1 =0$.
– Bates
Jul 15 '14 at 21:29




I think I can do it, if I assume that $H^1 =0$.
– Bates
Jul 15 '14 at 21:29




7




7




You don't need to assume $H^1=0$. But to prove $f$ is smooth, you have to assume that $phi$ is smooth. To prove that $df=phi$ at a point $p$, you can work in coordinates centered at $p$, in which case what you need to show is that $partial f/partial x^i(p) = phi_i(p)$ for each $i$, where the coordinate representation of $phi$ is $sum_i phi_i dx^i$. You can do this by carefully choosing a path whose last part travels along the $x^i$-axis in these coordinates. The proof is carried out in detail in my Introduction to Smooth Manifolds (2nd ed.), Theorem 11.42.
– Jack Lee
Jul 15 '14 at 22:19






You don't need to assume $H^1=0$. But to prove $f$ is smooth, you have to assume that $phi$ is smooth. To prove that $df=phi$ at a point $p$, you can work in coordinates centered at $p$, in which case what you need to show is that $partial f/partial x^i(p) = phi_i(p)$ for each $i$, where the coordinate representation of $phi$ is $sum_i phi_i dx^i$. You can do this by carefully choosing a path whose last part travels along the $x^i$-axis in these coordinates. The proof is carried out in detail in my Introduction to Smooth Manifolds (2nd ed.), Theorem 11.42.
– Jack Lee
Jul 15 '14 at 22:19












1 Answer
1






active

oldest

votes

















up vote
0
down vote













I assume we are discussing the problem in the context of differential geometry, so $M$ is a smooth manifold, $phi$ is a smooth 1-form. The smoothness of $f(x)=int_{x_0}^x phi$ is a local property so we should be able to prove that by applying fundamental theorem of calculus in $mathbf R ^n$.






share|cite|improve this answer





















    Your Answer





    StackExchange.ifUsing("editor", function () {
    return StackExchange.using("mathjaxEditing", function () {
    StackExchange.MarkdownEditor.creationCallbacks.add(function (editor, postfix) {
    StackExchange.mathjaxEditing.prepareWmdForMathJax(editor, postfix, [["$", "$"], ["\\(","\\)"]]);
    });
    });
    }, "mathjax-editing");

    StackExchange.ready(function() {
    var channelOptions = {
    tags: "".split(" "),
    id: "69"
    };
    initTagRenderer("".split(" "), "".split(" "), channelOptions);

    StackExchange.using("externalEditor", function() {
    // Have to fire editor after snippets, if snippets enabled
    if (StackExchange.settings.snippets.snippetsEnabled) {
    StackExchange.using("snippets", function() {
    createEditor();
    });
    }
    else {
    createEditor();
    }
    });

    function createEditor() {
    StackExchange.prepareEditor({
    heartbeatType: 'answer',
    convertImagesToLinks: true,
    noModals: true,
    showLowRepImageUploadWarning: true,
    reputationToPostImages: 10,
    bindNavPrevention: true,
    postfix: "",
    imageUploader: {
    brandingHtml: "Powered by u003ca class="icon-imgur-white" href="https://imgur.com/"u003eu003c/au003e",
    contentPolicyHtml: "User contributions licensed under u003ca href="https://creativecommons.org/licenses/by-sa/3.0/"u003ecc by-sa 3.0 with attribution requiredu003c/au003e u003ca href="https://stackoverflow.com/legal/content-policy"u003e(content policy)u003c/au003e",
    allowUrls: true
    },
    noCode: true, onDemand: true,
    discardSelector: ".discard-answer"
    ,immediatelyShowMarkdownHelp:true
    });


    }
    });














    draft saved

    draft discarded


















    StackExchange.ready(
    function () {
    StackExchange.openid.initPostLogin('.new-post-login', 'https%3a%2f%2fmath.stackexchange.com%2fquestions%2f868373%2fa-1-form-on-a-smooth-manifold-is-exact-if-and-only-if-it-integrates-to-zero-on-e%23new-answer', 'question_page');
    }
    );

    Post as a guest















    Required, but never shown

























    1 Answer
    1






    active

    oldest

    votes








    1 Answer
    1






    active

    oldest

    votes









    active

    oldest

    votes






    active

    oldest

    votes








    up vote
    0
    down vote













    I assume we are discussing the problem in the context of differential geometry, so $M$ is a smooth manifold, $phi$ is a smooth 1-form. The smoothness of $f(x)=int_{x_0}^x phi$ is a local property so we should be able to prove that by applying fundamental theorem of calculus in $mathbf R ^n$.






    share|cite|improve this answer

























      up vote
      0
      down vote













      I assume we are discussing the problem in the context of differential geometry, so $M$ is a smooth manifold, $phi$ is a smooth 1-form. The smoothness of $f(x)=int_{x_0}^x phi$ is a local property so we should be able to prove that by applying fundamental theorem of calculus in $mathbf R ^n$.






      share|cite|improve this answer























        up vote
        0
        down vote










        up vote
        0
        down vote









        I assume we are discussing the problem in the context of differential geometry, so $M$ is a smooth manifold, $phi$ is a smooth 1-form. The smoothness of $f(x)=int_{x_0}^x phi$ is a local property so we should be able to prove that by applying fundamental theorem of calculus in $mathbf R ^n$.






        share|cite|improve this answer












        I assume we are discussing the problem in the context of differential geometry, so $M$ is a smooth manifold, $phi$ is a smooth 1-form. The smoothness of $f(x)=int_{x_0}^x phi$ is a local property so we should be able to prove that by applying fundamental theorem of calculus in $mathbf R ^n$.







        share|cite|improve this answer












        share|cite|improve this answer



        share|cite|improve this answer










        answered Jun 6 '17 at 22:58









        wolfking01

        111




        111






























            draft saved

            draft discarded




















































            Thanks for contributing an answer to Mathematics Stack Exchange!


            • Please be sure to answer the question. Provide details and share your research!

            But avoid



            • Asking for help, clarification, or responding to other answers.

            • Making statements based on opinion; back them up with references or personal experience.


            Use MathJax to format equations. MathJax reference.


            To learn more, see our tips on writing great answers.





            Some of your past answers have not been well-received, and you're in danger of being blocked from answering.


            Please pay close attention to the following guidance:


            • Please be sure to answer the question. Provide details and share your research!

            But avoid



            • Asking for help, clarification, or responding to other answers.

            • Making statements based on opinion; back them up with references or personal experience.


            To learn more, see our tips on writing great answers.




            draft saved


            draft discarded














            StackExchange.ready(
            function () {
            StackExchange.openid.initPostLogin('.new-post-login', 'https%3a%2f%2fmath.stackexchange.com%2fquestions%2f868373%2fa-1-form-on-a-smooth-manifold-is-exact-if-and-only-if-it-integrates-to-zero-on-e%23new-answer', 'question_page');
            }
            );

            Post as a guest















            Required, but never shown





















































            Required, but never shown














            Required, but never shown












            Required, but never shown







            Required, but never shown

































            Required, but never shown














            Required, but never shown












            Required, but never shown







            Required, but never shown







            Popular posts from this blog

            Quarter-circle Tiles

            build a pushdown automaton that recognizes the reverse language of a given pushdown automaton?

            Mont Emei